Difference between revisions of "2021 Mock AMC 8 Problems/Problem 2"

(Created page with "==Problem== Aaron has a rectangular yard measuring <math>4</math> feet by <math>10</math> feet. How many <math>2</math> inch by <math>5</math> inch rectangular bricks can he f...")
 
(Solution 1)
 
(3 intermediate revisions by the same user not shown)
Line 2: Line 2:
 
Aaron has a rectangular yard measuring <math>4</math> feet by <math>10</math> feet. How many <math>2</math> inch by <math>5</math> inch rectangular bricks can he fit in his yard?
 
Aaron has a rectangular yard measuring <math>4</math> feet by <math>10</math> feet. How many <math>2</math> inch by <math>5</math> inch rectangular bricks can he fit in his yard?
  
<math>\mathrm{(A)}\ 4 \qquad\mathrm{(B)}\ 12 \qquad\mathrm{(C)}\ 24 \qquad\mathrm{(D)}\ 72 \qquad\mathrm{(E)}\ 144</math>
+
<math>\mathrm{(A)}\ 4 \qquad\mathrm{(B)}\ 72 \qquad\mathrm{(C)}\ 144 \qquad\mathrm{(D)}\ 288 \qquad\mathrm{(E)}\ 576</math>
  
 
==Solution 1==
 
==Solution 1==
 +
Since there are <math>12</math> inches in a feet, there are <math>2 \cdot 12</math> = <math>24</math> blocks every row. There are also <math>2 \cdot 12</math> = <math>24</math> blocks in each column. Therefore, we have <math>24 \cdot 24 = \boxed{\textbf{(E) } 576} \qquad</math>

Latest revision as of 16:55, 16 October 2021

Problem

Aaron has a rectangular yard measuring $4$ feet by $10$ feet. How many $2$ inch by $5$ inch rectangular bricks can he fit in his yard?

$\mathrm{(A)}\ 4 \qquad\mathrm{(B)}\ 72 \qquad\mathrm{(C)}\ 144 \qquad\mathrm{(D)}\ 288 \qquad\mathrm{(E)}\ 576$

Solution 1

Since there are $12$ inches in a feet, there are $2 \cdot 12$ = $24$ blocks every row. There are also $2 \cdot 12$ = $24$ blocks in each column. Therefore, we have $24 \cdot 24 = \boxed{\textbf{(E) } 576} \qquad$